Determining the Power of a Pole: f=\frac{z}{1-\cos z}

  • Thread starter Thread starter nhrock3
  • Start date Start date
  • Tags Tags
    Pole Power
Click For Summary
The discussion focuses on determining the power of the pole for the function f = z / (1 - cos z). It identifies singular points at z = 2πk and zero, with an emphasis on evaluating the pole's order. The user attempts to find the derivative g' and g'' but encounters indeterminate forms at z = 0. The conversation highlights the need to apply L'Hôpital's rule to resolve these forms and emphasizes that evaluating g'(0) directly is incorrect. The conclusion suggests that a Taylor series expansion could clarify the pole's nature, but the user prefers a derivative approach.
nhrock3
Messages
403
Reaction score
0
f=\frac{z}{1-\cos z}

the singular points are z=2pik and zero
i solved for z=2pik
and poles because there limit is infinity
now i want to determine te power of the pole
g=1/f=\frac{1-\cos z}{z}
g'=\frac{(-\sin z)z-(1-\cos z)}{z^2}
g'(0)=0/0
g''=\frac{-\sin z z^2 -(cos z -1)2z}{z^4}
g''(0)=0/0

the book says that its a first order pole

it should differ zero in order to be pole
 
Last edited:
Physics news on Phys.org
once again, try expanding the cosine as a taylor series about zero, should help you see what is happening
 
but i want to solve it this way
where did i go wrong in this way

i want to solve it by the derivative way
not by developing into a series
 
Last edited:
0/0 is an indeterminate form. You need to use the hospital rule to get an actual value for the limit.
 
but its not a limit
its a derivative
 
You can't just plug in 0 to evaluate g'(0) because you get an indeterminate form. You have to find the limit of g'(z) as z→0.
 
Question: A clock's minute hand has length 4 and its hour hand has length 3. What is the distance between the tips at the moment when it is increasing most rapidly?(Putnam Exam Question) Answer: Making assumption that both the hands moves at constant angular velocities, the answer is ## \sqrt{7} .## But don't you think this assumption is somewhat doubtful and wrong?

Similar threads

  • · Replies 9 ·
Replies
9
Views
2K
  • · Replies 7 ·
Replies
7
Views
1K
Replies
6
Views
2K
  • · Replies 2 ·
Replies
2
Views
2K
  • · Replies 2 ·
Replies
2
Views
2K
Replies
3
Views
2K
  • · Replies 1 ·
Replies
1
Views
1K
  • · Replies 3 ·
Replies
3
Views
2K
Replies
2
Views
2K
  • · Replies 15 ·
Replies
15
Views
4K